Let x , y , z be positive real numbers such that: x y z x ( y + 1 ) + y ( z + 1 ) + z ( x + 1 ) = 9 4 5 = 3 8 5 . If the minimum possible value of z + 2 y + 4 x can be written as b a , where a and b are coprime positive integers. What is the value of a + b ?
This section requires Javascript.
You are seeing this because something didn't load right. We suggest you, (a) try
refreshing the page, (b) enabling javascript if it is disabled on your browser and,
finally, (c)
loading the
non-javascript version of this page
. We're sorry about the hassle.
Can you please explain to as to why can't we use AM-Gm directly on x , y , z instead of doing it for ( x + 1 ) , ( y + 1 ) , ( z + 1 ) ?
Log in to reply
I think using AM-GM directly on x,y,z only considers the first equation and NOT the second equation.
Because the solution(x,y,z) we will obtain by using AM-GM directly on x,y,z will not satisfy the second equality relation.
Well-designed problem! If the conditions of the problem were instead x y z = 9 4 4 and x ( y + 1 ) + y ( z + 1 ) + z ( x + 1 ) = 3 8 6 , then the answer and method would be totally different.
x ( y + 1 ) + y ( z + 1 ) + z ( x + 1 ) = 3 8 5 → x y + x + y z + y + z x + z = 3 8 5 → x y z + x y + x + y z + y + z x + z = 1 3 3 0 → x y z + x y + x + y z + y + z x + z + 1 = 1 3 3 1 → ( x + 1 ) ( y + 1 ) ( z + 1 ) = 1 3 3 1 ( 1 )
By AM - GM inequality, 3 4 ( z + 1 ) + 2 ( y + 1 ) + ( x + 1 ) ≥ 3 4 ( z + 1 ) ∗ 2 ( y + 1 ) ∗ ( x + 1 ) 4 z + 4 + 2 y + 2 + x + 1 ≥ 6 3 ( z + 1 ) ( y + 1 ) ( x + 1 )
By (1) 4 z + 2 y + x + 7 ≥ 6 6 4 z + 2 y + x ≥ 5 9 4 4 z + 2 y + x ≥ 4 5 9 z + 2 y + 4 x ≥ 4 5 9
Therefore minimum value of z + 2 y + 4 x = 4 5 9 a + b = 5 9 + 4 = 6 3
But how do you know that this minimum value is attained? You have to use the original equation x y z = 9 4 5 somewhere! This is why I am not a huge fan of this problem; if you changed 945 and 385 to 944 and 386 (say) most people would get the same answer using the same method, but they would all be wrong. 945 was chosen judiciously so that the equality in the AM-GM inequality was attained.
Log in to reply
Yeah... Thought about that...
So, Suppose equality takes place in inequality, It implies that all 3 terms are equal, 4 ( z + 1 ) = 2 ( y + 1 ) = x + 1 = k ( L e t ) T h e n , 3 k = 6 6 k = 2 2 T h e r e f o r e , z = 2 9 , y = 1 0 , x = 2 1 x y z = 2 9 ∗ 1 0 ∗ 2 1 = 9 4 5
If xyz were 944 or any other value, then we would get 945 = 944, which is not possible. Equality will only be obtained when xyz = 945
Thanks for pointing that out
Wow, how is it that you realised to use the AM-GM inequality. Are there practice question for this topic. I also very confused by Patrick C.'s comment. All very interesting, I can follow your solution, just want to understand the problem solving!
Note that only mathematical statements should be included in the brackets. I have taken them out, to make your solution easier to read.
great good
Firstly, i saw that the minimum value can be easily got by using AM-GM on z + 2 y + 4 z but alaas! x y z is not a perfect cube and i could not get an accurate value. This is what i did then.
On expanding the second equation we get
x + y + z + x y + y z + z x = 3 8 5 .
It was indeed familiar expression. Just adding x y z + 1 to both sides gives us
1 + x + y + z + x y + y z + y z + x y z = 3 8 5 + 9 4 5 + 1 = 1 3 3 1 .
The left hand side can be factored, leaving us with
( x + 1 ) ( y + 1 ) ( z + 1 ) = 1 3 3 1 .
This is where a little trick has to be performed . Divide both sides by 8, so that we have
4 ( x + 1 ) 2 ( y + 1 ) ( z + 1 ) = 8 1 3 3 1
⟹ ( z + 1 ) ( 2 y + 2 1 ) ( 4 x + 4 1 ) = 2 3 1 1 3
Now we apply A M - G M inequality on the terms ( z + 1 ) , ( 2 y + 2 1 ) , ( 4 x + 4 1 )
We get
( z + 1 ) + ( 2 y + 2 1 ) + ( 4 x + 4 1 ) ≥ 3 3 ( z + 1 ) ( 2 y + 2 1 ) ( 4 x + 4 1 )
⟹ z + 2 y + 4 z + 4 7 ≥ 3 3 ( z + 1 ) ( 2 y + 2 1 ) ( 4 x + 4 1 ) = 3 × 2 1 1 = 2 3 3
⟹ z + 2 y + 4 z ≥ 2 3 3 − 4 7 = 4 5 9 and hence the answer 5 9 + 4 = 6 3
Q.E.D :)
i did it pretty much the same way, got (x+1)(y+1)(z+1) = 1331, then i wrote z +y/2 + x/4 as (4(z+1) + 2(y+1) + (x+1))/4 - (7/4) and then used AM-GM to minimize 4(z+1) + 2(y+1) + (x+1)
Adding the two equations gives ( x + 1 ) ( y + 1 ) ( z + 1 ) = 1 3 3 1 . By AM-GM,
z + 2 y + 4 x = 4 x + 1 + 2 y + 1 + ( z + 1 ) − 4 7 ≥ 3 3 8 ( x + 1 ) ( y + 1 ) ( z + 1 ) − 4 7 = 2 3 3 − 4 7 = 4 5 9 . Equality holds iff x = 2 1 , y = 1 0 , z = 4 . 5 . Thus, the minimum possible value of z + 2 y + 4 x is 4 5 9 and the answer is 5 9 + 4 = 6 3 .
Combine the first two equations and add one to get x + x y + y z + z + z x + z + x y z + 1 = ( x + 1 ) ( y + 1 ) ( z + 1 )
= 9 4 5 + 3 8 5 + 1 = 1 3 3 1 .
Rewrite what we want to minimize as ( z + 1 ) − 1 + 2 ( y + 1 ) − 2 1 + 4 ( x + 1 ) − 4 1
= ( z + 1 ) + 2 ( y + 1 ) + 4 ( z + 1 ) − 4 7 .
We know by AM-GM that ( z + 1 ) + 2 ( y + 1 ) + 4 ( z + 1 ) ≥ 3 ⋅ 3 ( z + 1 ) ⋅ 2 ( y + 1 ) ⋅ 4 ( z + 1 ) = 2 3 3 .
Equality is set when ( z + 1 ) = 2 ( y + 1 ) = 4 ( x + 1 ) and we get ( x , y , z ) = ( 2 1 , 1 0 , 4 . 5 ) . We see these numbers satisfy both equations.
2 3 3 − 4 7 = 4 5 9 . 5 9 + 4 = 6 3 .
Consider all the divisors of 945. Because it end with a 5, one of the factor must be at least 5. If one of the 3 variable, x ,y or z = 5 we end up with 189. Let's say x=5, then we have :
y z = 1 8 9
The divisors of 189 are (in pair)
1 ; 1 8 9 3 ; 6 3 7 ; 2 7 9 ; 2 1
Then we consider the last equation with those values. z is not divided by anything so giving it the smallest number is the most logical thing to do. All the cases can be written like this :
1 + 2 5 + 4 1 8 9 = 4 2 0 3 3 + 2 5 + 4 6 3 = 4 8 5 5 + 2 7 + 4 2 7 = 4 6 1 5 + 2 9 + 4 2 1 = 4 5 9
The smallest value is 59/4 which gives 63 when added together. The answer is 63.
x , y , z are real numbers, not integers, so this solution isn't rigorous.
Log in to reply
Indeed, but I didn't have any idea how to do it algebraically.
We add up the two equations...
⟹ ⟹ ⟹ ⟹ x ( y + 1 ) + y ( z + 1 ) + z ( x + 1 ) + x y z x y + x + y z + y + z x + z + x y z x y + y + z x + z + x y z + y z + x + 1 ( x + 1 ) ( y z + z + y + 1 ) ( x + 1 ) ( y + 1 ) ( z + 1 ) = = = = = 9 4 5 + 3 8 5 1 3 3 0 1 3 3 0 + 1 1 3 3 1 1 3 3 1 ⋅ ⋅ ⋅ ( 1 )
We apply the AM-GM inequality on ( 1 ) to get...
⟹ ⟹ ⟹ ⟹ ⟹ ⟹ ⟹ ∴ 3 ( x + 1 ) + 2 ( y + 1 ) + 4 ( z + 1 ) 3 ( x + 1 ) + 2 ( y + 1 ) + 4 ( z + 1 ) ( x + 1 ) + 2 ( y + 1 ) + 4 ( z + 1 ) x + 1 + 2 y + 2 + 4 z + 4 4 z + 2 y + x + 7 4 z + 2 y + x + 7 4 z + 2 y + x 4 4 z + 2 y + x z + 2 y + 4 x ≥ ≥ ≥ ≥ ≥ ≥ ≥ ≥ ≥ 3 ( x + 1 ) × 2 ( y + 1 ) × 4 ( z + 1 ) 3 8 ( x + 1 ) ( y + 1 ) ( z + 1 ) 2 × 3 3 ( x + 1 ) ( y + 1 ) ( z + 1 ) 6 3 ( x + 1 ) ( y + 1 ) ( z + 1 ) 6 3 1 3 3 1 = 6 3 1 1 3 ⋅ ⋅ ⋅ [ from ( 1 ) ] 6 × 1 1 = 6 6 6 6 − 7 = 5 9 4 5 9 4 5 9
Hence, the minimum possible value is: min ( z + 2 y + 4 x ) = b a = 4 5 9
Therefore, the required answer is: a + b = 5 9 + 4 = 6 3
As always, it is not sufficient to find the minimum value, but you must also show that it can hold.
In particular, because you are manipulating the equations, there is no reason why the equality condition must hold in the separate steps. For example,
If x and y are positive reals such that x 2 ≤ 9 and y 2 ≤ 1 , What is the maximum value of 2 x y ?
You cannot say that "Since x 2 + y 2 ≤ 1 0 hence 2 x y ≤ x 2 + y 2 = 1 0 . In fact, the maximum value is 6 , because the equality condition cannot hold.
Log in to reply
Also I'd like to point out that if we weren't concerned about the equality condition, we wouldn't have to perform AM-GM on ( z + 1 , 2 y + 1 , 4 x + 1 ) . Simply applying AM-GM inequality on ( z , 2 y , 4 x ) would suffice.
Adding the given equations, we obtain: x ( y + 1 ) + y ( z + 1 ) + z ( x + 1 ) + x y z = 9 4 5 + 3 8 5 ⟹ ( x + 1 ) ( y + 1 ) ( z + 1 ) = 9 4 5 + 3 8 5 + 1 = 1 3 3 1
Applying AM-GM inequality on the set ( z , 2 y + 1 , 4 z + 1 ) , we obtain: z + 2 y + 1 + 4 x + 1 ≥ 3 3 8 ( x + 1 ) ( y + 1 ) ( z + 1 ) ⟹ z + 2 y + 4 x + 2 1 + 2 1 ≥ 3 3 8 1 3 3 1 = 3 × 2 1 1 = 2 3 3 ⟹ z + 2 y + 4 x ≥ 2 3 3 − 2 1 − 4 1 = 4 5 9
Equality holds iff z + 1 = 2 y + 1 = 4 x + 1 . Solving this from the equation ( x + 1 ) ( y + 1 ) ( z + 1 ) = 1 3 3 1 , we obtain ( x , y , z ) = ( 2 1 , 1 0 , 2 9 ) . Plugging these values in the original equations, we see that they satisfy them. Hence equality holds for the set ( x , y , z ) = ( 2 1 , 1 0 , 2 9 ) , and the minimum possible value of z + 2 y + 4 x is 4 5 9 . Hence a = 5 9 , b = 4 , and a + b = 5 9 + 4 = 6 3 .
Two small typos:
We apply AM-GM on the set ( z + 1 , 2 y + 1 , 4 x + 1 ) .
Also, the sixth and seventh lines should have been:
z
+
2
y
+
4
x
+
1
+
2
1
+
4
1
≥
3
3
8
1
3
3
1
⟹
z
+
2
y
+
4
x
≥
2
3
3
−
1
−
2
1
−
4
1
=
4
5
9
The method of Lagrange Multipliers can be used to optimize f ( x , y , z ) = z + 2 y + 4 x . Instead of using two constraints, one can simply use a single constraint: one that describes the two simultaneously, i.e. the solution-equation.
x ( y + 1 ) + y ( z + 1 ) + z ( x + 1 ) = 3 8 5 = ( x + 1 ) ( y + 1 ) ( z + 1 ) − x y z − 1 3 8 5 = ( x + 1 ) ( y + 1 ) ( z + 1 ) − 9 4 5 − 1 1 3 3 1 = ( x + 1 ) ( y + 1 ) ( z + 1 )
Letting λ be the Lagrange multiplier, then we get the system
4 1 = ( x + 1 ) ( y + 1 ) λ 2 1 = ( z + 1 ) ( x + 1 ) λ 1 = ( x + 1 ) ( y + 1 ) λ
Dividing the first by the second, and the second by the third gives
2 1 = x + 1 y + 1 ⇒ x + 1 = 2 ( y + 1 ) 2 1 = y + 1 z + 1 ⇒ z + 1 = 2 y + 1
Substituting the expressions in terms of to the constraint 1 3 3 1 = ( x + 1 ) ( y + 1 ) ( z + 1 ) yields y + 1 = 1 1 . Hence, we can solve y = 1 0 , x = 2 1 and z = 2 9 . Therefore, we get z + 2 y + 4 x = 4 5 9 (One may check whether or not this is the minimum by using the Second-derivative test.)
Correction on the last paragraph: it should be "Substituting the expressions in terms of ( y + 1 ) to the constraint...".
By AM-GM inequality,
\frac{x}{4} + \frac{1}{4} + \frac{y}{2} + \frac{1}{2} + z+1 >= 3((\frac{x}{4}+\frac{1}{4})(\frac{y}{2}+\frac{1}{2})(z+1))^1/3 = 3((\frac{x+1}{4})(\frac{y+1}{2})(z+1))^1/3 = \frac{3}{8^1/3} * ((x+1)(y+1)(z+1))^1/3 = \frac{3}{2} (xyz + x(y+1)+y(z+1) + z(x+1) +1 )^1/3 = \frac{3}{2} (945 + 385 +1)^1/3 = \frac{3}{2} *1331^1/3 = \frac{3}{2} (11) = \frac{33}{2}
Hence \frac{x}{4} + \frac{y}{2} + z + \frac{7}{4} = \frac{33}{2} and \frac{x}{4} + \frac{y}{2} + z = \frac{59}{4} which means a=59 and b=4. Thus a+b = 63.
Problem Loading...
Note Loading...
Set Loading...
If we add two equations on the question, it will be similar to ( x + 1 ) ( y + 1 ) ( z + 1 ) .
We can figure out that ( x + 1 ) ( y + 1 ) ( z + 1 ) = 1 3 3 1 = 1 1 3 .
Because z + 2 y + 4 x can be changed into ( z + 1 ) + 2 ( y + 1 ) + 4 ( x + 1 ) − 4 7 , if we use AM-GM, ( z + 1 ) + 2 ( y + 1 ) + 4 ( x + 1 ) − 4 7 > = 3 ( 8 ( x + 1 ) ( y + 1 ) ( z + 1 ) ) 3 1 − 4 7 = 2 3 3 − 4 7 = 4 5 9
Hence, a = 5 9 , b = 4 , so the answer is 63.
(More specifically, z = 2 9 , y = 1 0 , x = 2 1 , when ( z + 1 ) = 2 ( y + 1 ) = 4 ( x + 1 ) )